Vous êtes sur la page 1sur 3

ECE 316 Probability Theory and Random Processes

Chapter 7 Solutions (Part 2)


Xinxin Fan
Problems
50. The joint density of X and Y is given by
f(x, y) =
e
x/y
e
y
y
, 0 < x < , 0 < y <
Compute E[X
2
| Y = y].
Solution. We rst nd the conditional PDF f
X|Y
(x | y) as follows:
f
X|Y
(x | y) =
e
x/y
e
y
/y
_

0
e
x/y
e
y
/y dx
=
1
y
e
x/y
, 0 < x < .
Hence, given Y = y, X is exponential with mean y, and so
E[X
2
| Y = y] = V ar(X | Y = y) + (E[X | Y = y])
2
= y
2
+ y
2
= 2y
2
.
51. The joint density of X and Y is given by
f(x, y) =
e
y
y
, 0 < x < y, 0 < y <
Compute E[X
3
| Y = y].
Solution. We rst nd the conditional PDF f
X|Y
(x | y) as follows:
f
X|Y
(x | y) =
e
y
/y
_
y
0
e
y
/y dx
=
1
y
, 0 < x < y.
Therefore, the conditional expectation can be computed with the denition.
E[X
3
| Y = y] =
_
y
0
x
3
f
X|Y
(x | y) dx =
_
y
0
x
3
1
y
dx =
y
3
4
.
53. A prisoner is trapped in a cell containing 3 doors. The rst door leads to a tunnel that returns
him to his cell after 2 days travel. The second leads to a tunnel that returns him to his cell
after 4 days travel. The third door leads to freedom after 1 day of travel. If it is assumed
1
that the prisoner will always select doors 1, 2 and 3 with respective probabilities .5, .3 and .2,
what is the expected number of days until the prisoner reaches freedom?
Solution. Let X denote the number of days until the prisoner is free, and let I denote the
initial door chosen. Then
E[X] = E[X | I = 1]P{I = 1} + E[X | I = 2]P{I = 2} + E[X | I = 3]P{I = 3}
= E[X | I = 1](.5) + E[X | I = 2](.3) + E[X | I = 3](.2)
= (2 + E[X])(.5) + (4 + E[X])(.3) + .2
Therefore, E[X] = 12.
56. The number of people who enter an elevator on the ground oor is a Poisson random variable
with mean 10. If there are N oors above the ground oor and if each person is equally likely
to get o at any one of these N oors, independently, of where the others get o, compute the
expected number of stops that the elevator will make before discharging all of its passengers.
Solution. Let
I
i
=
_
1 elevator stops at oor i
0 otherwise
.
Let X be the number of people that enter on the ground oor. Then we obtain
E
_
N

i=1
I
i
| X = k
_
=
N

i=1
E[I
i
| X = k] = N
_
1
_
N 1
N
_
k
_
,
where E[I
i
| X = k] = P{I
i
= 1 | X = k} = P{at least one person gets o at oor i | X =
k} = 1 P{no person gets o at oor i | X = k} = 1
_
N1
N
_
k
. Therefore, the expected
number of stops that the elevator will make can be computed as follows:
E
_
N

i=1
I
i
_
=

k=0
E
_
N

i=1
I
i
| X = k
_
P{X = k}
= N N

k=0
_
N 1
N
_
k
e
10
(10)
k
k!
= N Ne
10/N
= N(1 e
10/N
)
75. The moment generating function of X is given by M
X
(t) = exp{2e
t
2} and that of Y by
M
Y
(t) = (
3
4
e
t
+
1
4
)
10
. If X and Y are independent, what are E[XY ]?
Solution. Since X and Y are independent, we have E[XY ] = E[X] E[Y ]. Therefore, we
rst use the moment generating function to nd E[X] and E[Y ] as follows.
E[X] =
d
dt
M
X
(t)

t=0
= e
2e
t
2
2e
t

t=0
= 2,
E[Y ] =
d
dt
M
Y
(t)

t=0
= 10
_
3
4
e
t
+
1
4
_
9

3
4
e
t

t=0
=
15
2
.
Hence, we obtain E[XY ] = 2
15
2
= 15.
2
Theoretical Exercises
22. If Y = a + bX, show that
(X, Y ) =
_
+1 if b > 0
1 if b < 0
.
Solution. Note that Cov(X, Y ) = Cov(X, a+bX) = b Cov(X, X) = b V ar(X) and V ar(Y ) =
b
2
V ar(X). Therefore, we obtain
(X, Y ) =
b V ar(X)

b
2
V ar(X)
=
b
|b|
48. If Y = aX +b, where a and b are constants, express the moment generation function of Y in
term of the moment generating function of X.
Solution. M
Y
(t) = E[e
tY
] = E[e
t(aX+b)
] = e
tb
E[e
taX
] = e
tb
M
X
(ta).
49. The positive random variable X is said to be a lognormal random variable with parameters
and
2
if log(X) a normal random variable with mean and variance
2
. Use the normal
moment generating function to nd the mean and variance of a lognormal random variable.
Solution. Let Y = log(X). Since Y is normal with mean and variance
2
, it follows that
its moment generating function is
M(t) = E[e
tY
] = e
t+
2
t
2
/2
.
Hence, since X = e
Y
, we have that
E[X] = M(1) = e
+
2
/2
,
and
E[X
2
] = M(2) = e
2+2
2
.
Therefore, we get
V ar(X) = e
2+2
2
e
2+
2
= e
2+
2
(e

2
1).
50. Let X have moment generating function M(t), and dene (t) = log M(t). Show that

(t) |
t=0
= V ar(X).
Solution. Since (t) is equal to log M(t), we can nd the dierentiation of (t) as follows:

(t) = M

(t)/M(t),

(t) =
M(t)M

(t) (M

(t))
2
M
2
(t)
.
Finally, we obtain

(t) |
t=0
= E[X
2
] (E[X])
2
= V ar(X).
3

Vous aimerez peut-être aussi